Beziehungen zwischen den Seitenlängen und den Innenwinkelgrößen eines Dreiecks WS 11/12

Aus Geometrie-Wiki
Wechseln zu: Navigation, Suche

Inhaltsverzeichnis

Satz IX.2: (Der größeren Seite liegt der größere Winkel gegenüber)
Es sei \overline{ABC} ein Dreieck mit den schulüblichen Bezeichnungen.
\left| a \right| >\left| b \right| \Rightarrow \left| \alpha \right| > \left| \beta \right|
Beweis von Satz IX.2

Es sei \overline{ABC} ein Dreieck.

Voraussetzung:
\left| BC \right| > \left| AC \right| bzw. \left| a\right| > \left| b \right|

Behauptung:
\left| \alpha \right| > \left| \beta \right|

Die folgenden Hilfskonstruktionen liefern die Beweisidee (kommentieren Sie die Abbildungen und führen Sie den Beweis):

Seite winkel 01.png Seite winkel 02.png


Satz IX.3: (Dem größeren Winkel liegt die größere Seite gegenüber)
Es sei \overline{ABC} ein Dreieck mit den schulüblichen Bezeichnungen.
\left| \alpha \right| > \left| \beta \right|\Rightarrow \left| a \right| >\left| b \right|
Beweis von Satz IX.3

Übungsaufgabe

"Bierkastenbeweis"
Ich denke ich kann beweisen wie in der Vorlesung heute Morgen gewünscht.
Weiß nur nicht ob ich diesen Beweis in Wiki stellen soll, möchte es unseren Nachfolgern auch wieder die Chance lassen einen Kasten zu gewinnen.
Was meint ihr?--RicRic 16:58, 20. Jan. 2012 (CET)
Nachdem ich ja jetzt den Bierkasten habe, möchte ich bei erhalt jeden der sich dises Semester öfters akiv im Geowiki beteiligt hat, auf ein Bier einladen, die Tutoren natürlich auch und nur solange der Vorrat reicht :-)--RicRic 16:47, 28. Jan. 2012 (CET) Vor: \left| a \right| < \left| b \right|
Beh.: \left| \alpha  \right| < \left| \beta  \right|
Beweis:

Überschrift 1 Überschrift 2
(1) \exists \overline{CL} : L\in AB \wedge \ CL \perp \ AB Existenz und Eindeutigkeit des Lotes von einem Punkt auf eine Gerade
(2) \exists A' : A' \in AB \wedge \left| AL \right| = \left| A'L \right| \wedge A\neq A' Axiome Abstand, Lineal, (1)
(3) \overline{LC} \tilde {=} \overline{LC} trivial
(4) \overline{ALC} \tilde {=} \overline{A'LC} SWS, (1),(2),(3)
(5) \alpha \tilde {=} \alpha ' (4)
(6) \left| \beta \right|   > \left| \alpha '\right| schwacher Ausenwikelsatz
(7) \left| \beta \right|   > \left| \alpha \right| (5),(6)
--RicRic 16:47, 28. Jan. 2012 (CET)



Die Überlegung, Falls A' zwischen L und B liegen sollte, konnte ich magels Zeit leider nicht mehr vorstellen.


Überschrift 1 Überschrift 2
(1) \exists B': B' \in \overline{CA'} \wedge \left| CB' \right| = \left| CB \right| Axiom Lineal und Abstand
(2) \exists \overline{CB'B} ist Gelichschenklig (1)
(3) \angle \beta' \tilde {=} \angle B'BC Basiswikelsatz (2)
(4) \angle \beta' > \delta schwacher Ausenwikelsatz
(5)  \delta  + \alpha' = 180 nebenwinkel sind sulimentär
Wiederspruch ( 3),(4),(5) in einem Dreieck sie Summer zweier Innenwinkel immer kleiner als 180 sein muss Kollar zum schw Außenwineklsatz. Somit kann A nicht zwischen L und B liegen.

--RicRic 17:39, 28. Jan. 2012 (CET)


zweite Idee:


Vor: \left| a \right| < \left| b \right|
Beh.: \left| \alpha  \right| < \left| \beta  \right|
Beweis:

Überschrift 1 Überschrift 2
(1) \exists M: M \left| AM \right| = \overline{MB} \wedge M\in \overline{AB} Existenz und Eindeutigkeit des Mittelpunks einer Strecke
(2) \exists m: M\in m \wedge \ m \perp \ AB Existenz und Eindeutigkeit der Mittelsenkrechten einer Stecke, (1)
(3) Fall 1:\ m \cap \overline{AC} = \left\{ {S} \right\} Axiom von Pasch, Vor,2
(4) \overline{ALC} \tilde {=} \overline{A'LC} SWS, (1),(2),(3)
(5) \overline{ASB} ist gl. schenklig (3) Mittelsenkrechtenkriterum
(6) \alpha \tilde {=} \beta' (4) Basiswinkelsatz
(7) \left| \beta'  \right|  < \left| \beta'  \right| S im inneren von beta (4) Lemma, wenn die Endpunte einer Strecke auf den Schenkeln eines Winkels liegen liegt die gesamte Strecke im inneren des Winkels; Lemma, wenn ein Punkt im inneren eins Winkels liegt, liegt der gesamte Strahl im innenern dieses Winkels
\left| \alpha   \right|  < \left| \beta'  \right| (5),(6)

Fall2: m schneidet den Punkt C, daraus folgt nach dem Mitttelsenkrechtenkriterium, dass \left| a \right| = \left| b \right|
Wiederspruch zur Vorr.
Fall3: m schneidet die Strecke \overline{CB} darus folgt, A und C sind bezüglich m in der gleichen Halbebene, da eine Halbebene eine konvexe Punktmenge ist und M der Mittelpunkt von \overline{AB} und M der Mittelsenktrechten angehöt, müsste dies bedeuten: \left| a \right| > \left| b \right|
und ist somit ein Wiederspruch zur Vorr.--RicRic 17:04, 28. Jan. 2012 (CET)






Beweisidee: Ich weiß nicht, obs zu simple wäre, aber Herr Schnirch meinte, 2 Zeilen reichen.

Vor.:
\left| \alpha \right| > \left| \beta \right|

Beh.:
 \left| a \right| >\left| b \right|

Annahme:
 \left| a \right| <\left| b \right|  oder  \left| a \right| =\left| b \right| oBdA

Beweis Fall 1:
\left| \alpha neu \right| < \left| \beta neu \right|  Begründung: Annahme, Satz IX.2 (größere Seite, größerer Winkel)
Widerspruch zur Vor.

Beweis Fall 2:
\left| \alpha neu \right| = \left| \beta neu \right|  Begründung: Annahme, Basiswinkelsatz
Widerspruch zur Vor.
ps Ja halte ich für richtig.--RicRic 07:55, 23. Jan. 2012 (CET)